Math, asked by shobha1211983, 8 months ago

Please help me solve this question​

Attachments:

Answers

Answered by sahilarekar16
0

Step-by-step explanation:

b)10√13 is the correct answer for your question

Answered by deepakbhaskar027
1

Answer:

+-10√13

Step-by-step explanation:

let a/b=x then b/a will be 1/x

x-1/x=3

squaring both side

(x-1/x)^2=(3)^2

(x)^2+(1/x)^2-2x*1/x=9

x^2+(1/x)^2-2=9

adding 1 both sides,we get

x^2+(1/x)^2-1=10....(1)

now

(x-1/x)^2 in term of 4

such that

(x+1/x)^2-4*x*1/x=9

(x+1/x)^2=13

(x+1/x)=+-√13..(2)

now

we know that

x^3+1/x^3=(x+1/x)[x^2-x*1/x+(1/x)^2]

putting the value from equation 1 and2

we get

x^3+(1/x)^3=+-10√13

now finally

(a/b)^3+(b/a)^3=+-10√13

hope it's help

Similar questions